The Makespan is 790.
The Flow Time for Each Job is 70, 210, 410, 530, 690, 790.
The overall Average Flow Time is 450.
To compute the makespan, flow time for each job, and overall average flow time, we'll use the First-Come-First-Served (FCFS) scheduling algorithm.
Given the processing times for each job:
Job: 1 2 3 4 5 6
Time: 70 140 200 120 160 100
Makespan:
The makespan is the total time required to complete all jobs. It is the sum of the processing times for all jobs.
Makespan = 70 + 140 + 200 + 120 + 160 + 100 = 790
Flow Time for Each Job:
The flow time for a job is the time from when the job arrives until it is completed. For FCFS, it is simply the cumulative processing time of all the previous jobs plus the processing time of the current job.
Job: 1 2 3 4 5 6
Time: 70 140 200 120 160 100
Flow Time: 70, 210, 410, 530, 690, 790
Overall Average Flow Time:
To calculate the average flow time, we sum up the flow times for all the jobs and divide by the number of jobs.
Average Flow Time = (70 + 210 + 410 + 530 + 690 + 790) / 6 = 450
Therefore, Makespan: 790
Flow Time for Each Job: 70, 210, 410, 530, 690, 790
Overall Average Flow Time: 450
Learn more about First-Come-First-Served scheduling algorithm here
https://brainly.com/question/2260537
#SPJ4
Find the area of a square if its side length is:
1. 1/5 cm 2. 3/7 units 3. 11/8 inches 4. 0.1 meters 5. 3.5 cm
Answer:
1. 1/25 cm^2
2. 9/49 u^2
3. 121/64 in^2
4. 0.01 m^2
5. 12.25 cm^2
Step-by-step explanation:
Take each of the values and square them
EXAMPLE:
(1/5)^2 = 1^2/5^2=1/25
Also, INCLUDE UNITS
Solve tan 9°
=x/2.6
Give your answer to 2 d.p.
Answer:
x= 0.41
Step-by-step explanation:
tan 9° =\(\frac{x}{2.6}\)
x= 2.6 tan9° (cross multiply)
x= 0.41 (solve trignomatic function)
Please Help due today
Answer:
x = 9
Step-by-step explanation:
You want the length of the hypotenuse in the right triangle with one hypotenuse segment of length 4 and altitude to the hypotenuse of length 2√5.
Geometric meanThe altitude of the triangle is the geometric mean of the segments of the hypotenuse:
2√5 = √(4·(x-4))
4·5 = 4·(x -4) . . . . . . square both sides
5 = x -4 . . . . . . . divide by 4
9 = x . . . . . . . add 4
The length of x is 9 units.
<95141404393>
How is this wrong? Please explain to me.
Answer:
Wrong values are plotted in the question statement graph.
Please find correct graph.
Step-by-step explanation:
Given that:
\(y =x+3\) and \(-1<x<5\)
i.e. we can put a value of x in the equation which is greater than -1 and lesser than 5.
OR
we can say, the points where x = -1 and x = 5 will not be included in the graph (they will be shown as hollow circle).
Now, let us put the values one by one:
x = -1, y = -1 + 3 =2
x = 0, y = 0 + 3 = 3
x = 1, y = 1 + 3 = 4
x = 2, y = 2 + 3 = 5
x = 3, y = 3 + 3 = 6
x = 4, y = 4 + 3 = 7
x = 5, y = 5 + 3 = 8
Therefore, the table of values become:
\(\begin{center}\begin{tabular}{ c c } x & y\\ -1 & 2\\ 0 & 3 \\ 1 & 4 \\ 2 & 5 \\ 3 & 6 \\ 4 & 7 \\5 & 8\\\end{tabular}\end{center}\)
Points (-1, 2) and (5, 8) will not be included in the graph.
Please find attached corrected graph.
Identify the characters of series below. nvž enn |||-) En=12 100 1-) Σπίο 3* 2"-1 ||-) En=2 n A) I Convergent, II Divergent, III Convergent B) I Convergent, Il Convergent, III Divergent C) I Convergent, II Convergent, III Convergent D) I Divergent, Il Divergent, III Divergent E) I Divergent, II Divergent, III Convergent
Based on the information, we can determine convergence or divergence of series.The given options do not provide a clear representation of potential outcomes.It is not possible to select correct option.
The given series is "nvž enn |||-) En=12 100 1-) Σπίο 3* 2"-1 ||-) En=2 n". In the series, we have the characters "nvž enn |||-)" which indicate the series notation. The characters "En=12 100 1-" suggest that there is a summation of terms starting from n = 12, with 100 as the first term and a common difference of 1. The characters "Σπίο 3* 2"-1 ||-) En=2 n" indicate another summation, starting from n = 2, with a pattern involving the operation of multiplying the previous term by 3 and subtracting 1.
To learn more about convergence click here : brainly.com/question/32549533
#SPJ11
Which set of measurements can represent the lengths of a triangle's sides?
Classify each set of lengths as a Triangle or Not a triangle.
The correct statements regarding whether the set of measures can represent a triangle are given as follows:
2 cm, 4 cm and 5 cm: Triangle.2 cm, 2 cm and 5 cm: Not a triangle.2 cm, 5 cm and 6 cm: Triangle.1 cm, 2 cm and 5 cm: Not a triangle.When the set of measurements can represent a triangle?A set of measurements can represent a triangle if the sum of the two smaller measurements is greater than the greater measures.
Then the lengths in this problem are classified as follows:
2 cm, 4 cm and 5 cm: Triangle, as 2 + 4 > 5.2 cm, 2 cm and 5 cm: Not a triangle, as 2 + 2 < 5.2 cm, 5 cm and 6 cm: Triangle, as 2 + 5 > 6.1 cm, 2 cm and 5 cm: Not a triangle, as 1 + 2 < 3.More can be learned about set of measurements and triangles at https://brainly.com/question/24432457
#SPJ1
1. What is the value of this expression when c = 4?
4C+ 3c- 2c
Answer:
20
Step-by-step explanation:
4(4)+3(4)-2(4)=20
Find
dx
dy
for the following equations. a. y=−x
2
+5x+2 b. y=2x
2
−8x+10 c. y=−2x
2
+9x−1
the derivative of y = -\(x^2\) + 5x + 2 is dy/dx = -2x + 5. . For y = 2\(x^2\) - 8x + 10 the dy/dx = 4x - 8. for y = -2\(x^2\)+ 9x - 1 the dy/dx = -4x + 9, after differentiation.
a. To find the derivative of y = -\(x^2\) + 5x + 2, we differentiate each term with respect to x. The power rule states that for a term of the form \(x^n,\) the derivative is n*\(x^(n-1)\). Applying this rule, we get:
dy/dx = -2x + 5
b. For y = 2\(x^2\)- 8x + 10, we again differentiate each term using the power rule:
dy/dx = 4x - 8
c. Lastly, for y = -2\(x^2\)+ 9x - 1, we differentiate each term:
dy/dx = -4x + 9
In each case, we obtain the derivative of y with respect to x. The resulting derivatives represent the instantaneous rate of change of y with respect to x at any given point on the curve. They also indicate the slope of the tangent line to the curve at that point. By finding the derivatives, we gain insight into the behavior and characteristics of the functions, such as the direction of increasing or decreasing values and the presence of maximum or minimum points.
Learn more about curve here:
https://brainly.com/question/32535381
#SPJ11
how many different three-letter strings can be formed from the letters a, b, c, d, e (repeats allowed) if the string must contain either all vowels or all consonants?
8 strings can be formed with all vowels from a, b, c, d, e and 27 strings can be formed with all consonants from a, b, c, d and e.
What is permutation and combination?
A permutation is an orderly arrangement of things or numbers.
Combinations are a means to choose items or numbers from a collection or set of items without worrying about the items' chronological order.
The amount of two-letter words that can be created using the letters in a word, like GREAT, is an illustration of permutations: ⁵P₂ = 5!/(5-2)!
How many different ways we may write the words utilizing the vowels in the word GREAT is an example of a combination: ⁵C₂ = 5!/[2! × (5-2)!]
Given, the letters that can be used to create the required three letter word in question: a, b, c, d, e.
Total number of letters that can be used = 5
Total number of vowels in the available letters to be used = 2 ('a' and 'e')
Total number of consonants in the available letters to be used = 3 ('b', 'c' and 'd')
Case 1: A three letter word having all vowels where letters can be repeated
Here, since two letters are to put in three empty positions, the total number of ways to form this word = 2³ = 2 × 2 × 2 = 8
Case 2: A three letter word having all consonants where letters can be repeated
Here, since three letters are to put in three empty positions, the total number of ways to form this word = 3³ = 3 × 3 × 3 = 27
Thus, 8 strings can be formed with all vowels from a, b, c, d, e and 27 strings can be formed with all consonants from a, b, c, d and e.
To learn more about permutation and combination, tap on the link below:
https://brainly.com/question/11732255
#SPJ4
What is the decimal of thirty five hundred thousandths?
Answer:
0.0035
Step-by-step explanation:
In front of the decimal point is the ones place. After that is the tenths place, hundredths place, and then thousandths place.
Which of the following statements are true regarding the margin of error? Select all correct options. A large confidence level requires a small value of 2*, Being more confident increasing confidence level) will yield a larger margin of error The margin of error is not influenced by the sampling distribution's standard deviation Selecting a larger SRS from the population will yield a smaller margin of error.
The following statements are true regarding the margin of error: Being more confident (increasing confidence level) will yield a larger margin of error and selecting a larger SRS from the population will yield a smaller margin of error.
A margin of error (MOE) is the degree of inaccuracy in estimating a population's true proportion or mean by analyzing a sample dataset. It represents the uncertainty or confidence level of a poll's findings. It is a critical element in political polls, scientific studies, and marketing research. The accuracy of the findings is expressed in the margin of error, which is given as a percentage. The standard deviation of the sampling distribution, not the standard deviation of the population, determines the margin of error. As a result, selecting a larger sample size, the standard deviation decreases, and the margin of error decreases. Therefore, selecting a larger SRS from the population will yield a smaller margin of error. Confidence intervals are used to determine the margin of error, and increasing the confidence level will result in a larger margin of error. As a result, being more confident (increasing confidence level) will yield a larger margin of error.
Thus, the correct options are (B) and (D). Option (A) is incorrect because a large confidence level requires a large value of 2*. Option (C) is incorrect because the margin of error is influenced by the standard deviation of the sampling distribution.
Learn more about margin of error visit:
brainly.com/question/29419047
#SPJ11
Translate the following into math expression: " 3 more than a number is 5 less than double this number ."
Answer:
3 + n = 2n - 5
Step-by-step explanation:
let 'n' equal 'a number'
'more than' implies 'addition', so 3 + n
'is' in math implies 'equals', so 3 + n =
'double the number' means to multiply it by 2
'less than' implies 'subtraction'
So final result is:
3 + n = 2n - 5
The Sugar Sweet Company is going to transport its sugar to market. It will cost $3500 to rent trucks plus $175 for each ton of sugar transported. The total cost,
C (in dollars), for transporting n tons is given by the following.
C=175n+3500. Question A: what’s is the total cost of transporting 13 tons? Question B: if the total cost is $7875, how many tons is the company transporting
Part A
\(C=175(13)+3500=\boxed{\$5775}\)
Part B
\(7875=175n+3500 \\ \\ 4375=175n \\ \\ n=\boxed{25}\)
A theater has 1,464 seats. The seats are arranged into 62 equal-sized "regular" sections plus one "premium" front-row section. How many seats are in a regular section? How many seats are in the premium front-row section? Explain.
In the theater, that have 1,464 seats.
1426 seats are in "regular" sections
38 seats are "premium" front-row section
How to find the number of seats in the "regular" sectionsThe seat arrangement is solved by division. In this case the 62 equal spaced is the divisor while the number of seats is the in each row is the quotient
The division is as follows
1464 / 62
= 23 19/31
The number of seats in the regular section is 23 * 62 = 1426
The remainder will be arranged in premium front row
using equivalent fractions
19 / 31 = 38 / 62
the remainder is 38 and this is the seat for the premium front row section
OR 1464 - 1426 = 38 seats
Learn more about division here:
https://brainly.com/question/29401139
#SPJ1
You and your sister are selling cookies to help raise money for your field trip. You start out with $24 and sells each bag of cookies, c, for $3. Your sister doesn’t start out with any money but sells her bags of cookies for $5 each. How many bags of cookies must they sell in order for them to raise the same amount of money?
Answer:
12 bags of cookies.
Step-by-step explanation:
Since you already start out with $24, you will have a y-intercept of 24. Your slope will be 3, since each bag sells for $3.
Your equation will be y = 3c + 24.
Your sister does not start out with money, so she will have a y-intercept of 0. Her slope will be 5, as each bag sells for $5.
Her equation will be y = 5c.
Since y = y, you can set the two equations equal to each other.
3c + 24 = 5c
5c = 3c + 24
Subtract 3c from both sides
2c = 24
Divide both sides by 2
c = 12
So, they must sell 12 bags of cookies to raise the same amount of money, $60. Yum!
Hope this helps!
factor completely 8y2 + 6y + 1
Answer:
Step-by-step explanation:
8y²+6y+1
=8y²+4y+2y+1
=4y(2y+1)+1(2y+1)
=(2y+1)(8y+1)
If G is the midpoint of FH , FG = 14x + 25 and GH = 73 -2x, find FH
Answer:
146 units
Step-by-step explanation:
midpoint which means both lengths equal each other
GH=FG
14x+25=73-2x
12x=48
x=4
FH would therefore= both lines added together
14(4)+25+73-2(4)
=146 units
suppose you have an uneven 6-sided die where the numbers 1-5 are equally likely to occur, and the expected value of the entire die is 4. what is the probability of getting a 6?
The likelihood of striking a 6 on the this unbalanced die is therefore 1, or 100%.
What does value mean to you?A reasonable transaction result in money, products, or services. 2.: The monetary value of anything. 3.: value, usefulness, or significance in relation to anything else
As the die has only 6 sides, the probability of rolling a 6 is denoted by P(6). Let's use the formula of expected value to find P(6).
The expected value of the die is given by:
E(X) = (1 + 2 + 3 + 4 + 5 + 6)/6 = 21/6 = 3.5
We know that the expected value of the entire die is 4. Therefore, the expected value of the additional bonus for rolling a 6 is:
E(Y) = 4 - E(X) = 4 - 3.5 = 0.5
Since the bonus value can only be obtained by rolling a 6, the probability of getting a 6 is equal to the probability of obtaining the bonus value of 0.5. Hence, we have:
P(6) = P(Y = 0.5) = P(X = 1) + P(X = 2) + P(X = 3) + P(X = 4) + P(X = 5)
Since the numbers 1-5 are equally likely to occur, we have:
P(X = 1) = P(X = 2) = P(X = 3) = P(X = 4) = P(X = 5) = 1/5
Therefore,
P(6) = P(X = 1) + P(X = 2) + P(X = 3) + P(X = 4) + P(X = 5) = 5/5 = 1
To know more about Value visit:
https://brainly.com/question/24170327
#SPJ1
X +4
2 - 16
lim
X→-4 X
I'm going to assume you meant to say
\(\displaystyle \lim_{x\to-4} \frac{x+4}{x^2-16}\)
Factorize the denominator to reveal a removable discontinuity in the limand:
\(\dfrac{x+4}{x^2-16} = \dfrac{x+4}{(x+4)(x-4)}\)
In the limit, we're considering values of x *near* -4, and not x = -4 itself. So we can cancel the factors of x + 4 and we're left with
\(\displaystyle \lim_{x\to-4} \frac{x+4}{x^2-16} = \lim_{x\to-4} \frac1{x-4} = \frac1{-4-4} = \boxed{-\frac18}\)
A college conducts a common test for all the students. For the Mathematics portion of this test, the scores are normally distributed with a mean of 502 and a standard deviation of 115. Based on this information, what percentage of students would be expected to score between 400 and 590
Using the normal distribution, it is found that 58.97% of students would be expected to score between 400 and 590.
Normal Probability DistributionThe z-score of a measure X of a normally distributed variable with mean \(\mu\) and standard deviation \(\sigma\) is given by:
\(Z = \frac{X - \mu}{\sigma}\)
The z-score measures how many standard deviations the measure is above or below the mean. Looking at the z-score table, the p-value associated with this z-score is found, which is the percentile of X.The mean and the standard deviation are given, respectively, by:
\(\mu = 502, \sigma = 115\)
The proportion of students between 400 and 590 is the p-value of Z when X = 590 subtracted by the p-value of Z when X = 400, hence:
X = 590:
\(Z = \frac{X - \mu}{\sigma}\)
\(Z = \frac{590 - 502}{115}\)
Z = 0.76
Z = 0.76 has a p-value of 0.7764.
X = 400:
\(Z = \frac{X - \mu}{\sigma}\)
\(Z = \frac{400 - 502}{115}\)
Z = -0.89
Z = -0.89 has a p-value of 0.1867.
0.7764 - 0.1867 = 0.5897 = 58.97%.
58.97% of students would be expected to score between 400 and 590.
More can be learned about the normal distribution at https://brainly.com/question/27643290
#SPJ1
Please help
The number of high school seniors graduating from a certain high school increases each year. The equation y = 21.8571x + 309.3333 is the least squares regression line for the number of high school seniors, y, graduating from a high school x years since 2010. For each subsequent year, about how many more seniors graduate from the high school? Round to the nearest whole number
Answer:
22
Step-by-step explanation:
The 309.3333 is the base amount of seniors that graduate each year, the x is the number of years, so the 21.8571 is the number of students they increase by. This answer is 100% I took the final and got it right. Hope this helps!! :)
Answer:
22
Step-by-step explanation:
Alexander is a stockbroker. He earns 13% commission each week. Last week, he sold $7,500 worth of stocks. How much did he make last week in commission? If he averages that same amount each week, how much did he make in commission in 2011? Last week, he made $ in commission.
EXPLANATION
Let's see the facts:
%Earns= 13% commission/week
Last week sold = $7,500
In order to obtain the last week earns in commission we need to multiply % rate of commission by the last week sold as shown as follows:
$Earns = 0.13 (decimal form)*$7,500
Earns = $975
Last week, he made $975 in commission.
If he averages that same amount each week, he should make $975* 52(number of weeks per year) = $50,700/year
Mark bought 3 shirts at a clothing store. If he paid a total of $15.00 for 2 shirts and the arithmetic mean cost of the 3 shirts was $8.00, how much did mark pay for the third shirt ?
Answer:
from what I know he paid.$ 22.50
explanation
2 shirts cost $ 15.00
what about one shirt $ 1500
2
one shirt costs $750
what about 3 shirts $750x3= $2250
total was $ 2250 so he paid $750 since one shirt costs $750
A PE class has 48 students. One group of 24 students chooses to play volleyball. The teacher then divides the
remaining students into as many groups of 5 as possible to shoot baskets. After that the remaining students (not
on a volleyball or basketball team) climb the rock wall.
How many students climb the rock wall?
students
Subtraction is a mathematical operation that reflects the removal of things from a collection. The number of students who climbs the rock wall is 4.
What is subtraction?Subtraction is a mathematical operation that reflects the removal of things from a collection. The negative symbol represents subtraction.
The total number of students in the class is 48, out of these 48 students, 24 students choose to play volleyball. Therefore, the remaining students are 24 (48-24).
Students remaining after volleyball
= Total number of students - Number of students who play volleyball
= 48 - 24 = 24
Now, The teacher then divides the remaining students into as many groups of 5 as possible to shoot baskets. Therefore, the teacher divides the remaining 24 students into 4 groups of 5 people. After this, the number of students that are remaining is 4(24-20).
Students remaining after basketball
= Total number of students - Number of students who shoot basketball
= 24 - (4x5)
= 24 - 20 = 4
Further, the remaining students go for climbing the rock wall.
Hence, the number of students who climbs the rock wall is 4.
Learn more about Subtraction:
https://brainly.com/question/1927340
#SPJ1
a certain basketball player makes a foul shot with probability 0.45. what is the probability that (a) his first basket occurs on the sixth shot; (b) his first and second baskets occur on his fourth and eighth shots, respectively?
The probability of making a basket on the sixth shot is 0.075. The probability that the first and second baskets occur on the fourth and eighth shots is 0.031.
For the first shot to be on the sixth trial, five earlier shots must have failed.
The probability of a missed basket is 1 - 0.45 = 0.55.
The probability of five missed baskets in a row is (0.55)5 = 0.166.
The probability of making a basket on the sixth shot is 0.45.
Hence, the probability of making a basket on the sixth shot is 0.45 * 0.166 = 0.075.
There are a few possible ways to get the first and second baskets to happen on the fourth and eighth shots, respectively.
In the end, either of the following two paths will work:
4 failures, then 2 successes, then 2 failures
3 failures, then 1 success, then 1 failure, then 1 success, then 2 failures
Using the multiplication rule, we get the following probabilities for each course:
P(4 failures, 2 successes, 2 failures) = (0.55)4 * (0.45)2 * (0.55)2
= 0.018
P(3 failures, 1 success, 1 failure, 1 success, 2 failures) = (0.55)3 * (0.45) * (0.55) * (0.45) * (0.55)2
= 0.013
The probabilities of the two possible routes must be summed up to arrive at the answer.
Hence, the probability that the first and second baskets occur on the fourth and eighth shots, respectively, is 0.018 + 0.013 = 0.031.
Learn more about probability here: https://brainly.com/question/24756209
#SPJ11
Write the compound inequality: " A number n is less than or equal to -7 or greater than 12
Let the number be n, then -1<3n+5<38 or -1≤3n+5≤38 depending on what is meant by “between”. We’ll assume the former.
-1<3n+5<38, subtract 5 throughout: -6<3n<33, divide through by 3: -2<n<11, shows the number to be in the range (-2,11) or possibly [-2,11].
R950 is divided between Ann, Jill, Agnes Ann gets R100 more than Jill, Jill gets R50 more than Agnes. How much did Agnes get
Jamal’s test scores are 84, 88, 92, and 94. Natalia’s test scores are 86, 88, 90, 92. Who has the greater test average and who has the most consistent test scores?
Jamal’s test scores are 84, 88, 92, and 94. Natalia’s test scores are 86, 88, 90, 92. Jamal has a greater test average than Natalia. Natalia has more consistent test scores than Jamal.
To find the test average, you add up all the test scores and divide by the number of test scores.
Jamal's average: (84+88+92+94) / 4 = 89.50
Natalia's average: (86+88+90+92) / 4 = 89
Jamal has a greater test average.
To find the consistency of test scores, you can use the range, which is the difference between the highest and lowest scores.
Jamal's range: 94-84=10
Natalia's range: 92-86 = 6
Natalia has the most consistent test scores, as the range of her scores is smaller than Jamal's.
To know more on mean
https://brainly.com/question/23263573
#SPJ4
Jared also gathered data from a competing real estate broker's office. The competitor's sales were equal to the average
number of sales 8 months before Jared began gathering data and again 2 and 6 months after he began gathering data.
The relative minimum number of sales was lower than the relative minimum number of sales of Jared's office.
Use the sliders for a, b, c, and d to create a graph that could model the competitor's number of sales.
Real estate is any immovable property of this kind that has a stake in a piece of land, a building, or housing in general. It also includes the natural resources on the land, such as water, crops, minerals, and minerals.
How does this occur?
from a company real estate broker's office. The competitor's sales were equal to the average number of sales 8 months before jured began gathering data and again 2 and 6 months after he began gathering data. The relative minimum number of sales was lower than the relative minimum number of sales of Jared's office.
Use the sliders for a, b, c, and d to create a graph that could model the competitor's number of sales.
Graph is given below in Image.
Learn more about real estate link below
https://brainly.com/question/1534216
#SPJ1
please find what number x is
Step-by-step explanation:
Hey there!
Given;
The triangle is a right angled triangle, whose one side is 26 and another side is X. It has also one angle that is 41°.
Now, taking reference angle as angle 41°. We get;
Perpendicular= 26
Base = X
and Reference angle = 41°.
We know that the ratio of tan is p/b. So, let's use this ratio.
\( \tan(41°) = \frac{26}{x} \)
\(0.869 \times x = 26\)
\(x = \frac{26}{0.869} \)
Therefore, The value of x is 29.90.
Hope it helps...